Đến nội dung

Hình ảnh

[TOPIC] ÔN THI BẤT ĐẲNG THỨC $\boxed{\text{THPT CHUYÊN}}$ LỚP $10$ năm $2018-2019$

bất đẳng thức holder cosi bunhiacopxki

  • Chủ đề bị khóa Chủ đề bị khóa
Chủ đề này có 318 trả lời

#281
MoMo123

MoMo123

    Sĩ quan

  • Điều hành viên THCS
  • 334 Bài viết

Khuấy đảo topic lại nào

$\boxed{\text{Bài 125}}$ Cho a,b,c là các số thực dương

Chứng minh $$\frac{2a}{b+c}+\frac{2b}{a+c}+\frac{2c}{a+b} \geq 3+\frac{(a-b)^2+(b-c)^2+(c-a)^2}{(a+b+c)^2}$$

$\boxed{\text{Bài 126}}$ Cho a,b,c  là các số thực dương

Chứng minh $$\frac{2a^2+ab}{(a+b+\sqrt{ca})^2}+\frac{2b^2+bc}{(b+c+\sqrt{ab})^2}+\frac{2c^2+ca}{(c+a+\sqrt{bc})^2} \geq 1$$

 

$\boxed{\text{Bài 127}}$ Cho a,b,c là các số thực dương

Chứng minh rằng $$(1+\frac{a}{b})(1+\frac{b}{c})(1+\frac{c}{a}) \geq 2(1+\frac{a+b+c}{\sqrt[3]{abc}})$$

Mình xin đưa ra 1 lời giải khác của bài 127.

Đặt $x=\frac{a}{\sqrt[3]{abc}}$ $y=\frac{b}{\sqrt[3]{abc}}$ ; $z=\frac{c}{\sqrt[3]{abc}}$

$\Rightarrow \frac{x}{y}=\frac{a}{b}$ ;$ \frac{y}{z}=\frac{b}{c}$; $\frac{z}{x} =\frac{c}{a}$ và xyz =1

Bất đẳng thức cần Chứng minh tương đương

$$(x+y)(y+z)(z+x) \geq 2(1+x+y+z)$$

Ta có: $$(x+y)(y+z)(z+x)=(x+y+z)(xy+yz+zx)  -xyz =(x+y+z)(xy+yz+zx) -1$$

Tức ta cần chứng minh $$(x+y+z)(xy+yz+zx-2) \geq 3(1)$$

Ta có $$x+y+z \geq 3\sqrt[3]{xyz}=1$$

$$xy+yz+zx \geq 3\sqrt[3]{a^2b^2c^2}=3$$ Nên (1) được chứng minh



#282
Le Hoang Anh Tuan

Le Hoang Anh Tuan

    Binh nhất

  • Thành viên mới
  • 27 Bài viết

Bài 129: $a, b, c \geqslant 0, a+b+c=1$

Chứng minh: $\frac{1}{1+6a^{2}} + \frac{1}{1+6b^{1}}+\frac{1}{1+6c^{2}} \geqslant \frac{9}{5}$



#283
tr2512

tr2512

    Thượng sĩ

  • Thành viên
  • 272 Bài viết

Bài 129: $a, b, c \geqslant 0, a+b+c=1$

Chứng minh: $\frac{1}{1+6a^{2}} + \frac{1}{1+6b^{1}}+\frac{1}{1+6c^{2}} \geqslant \frac{9}{5}$

 Giả sử $c=$min$\{a,b,c\}$
 Nếu $c \ge \frac{1}{12}$, ta sẽ đi chứng minh:
 $$\frac{1}{6c^2+1} \ge -\frac{36}{25}(c-\frac{1}{3})+\frac{3}{5}$$
 $$ \Leftrightarrow \frac{2(3c-1)^2(12c-1)}{25(6c^2+1)} \ge 0$$
Bất đẳng thức trên luôn đúng do $c \ge \frac{1}{12} $
Lập 2 bất đẳng thức tương tự thu được điều phải chứng minh.
Nếu $c \le \frac{1}{12}$
Ta sẽ đi chứng minh:
$$ \frac{1}{6a^2+1} \ge -\frac{24}{25}(a-\frac{1}{2}) +\frac{2}{5}$$
$$ \Leftrightarrow  \frac{3(2a-1)^2(12a+1)}{25(6a^2+1)} \ge 0 $$
Bất đẳng thức này luôn đúng.
Lập 2 bất đẳng thức như vậy với $a, b$ thu được:
$$\frac{1}{6a^2+1}+\frac{1}{6b^2+1} \ge \frac{24}{25}c+\frac{4}{5}$$
Vậy ta chỉ cần chứng minh:
$$\frac{24c}{25}+\frac{4}{5}+\frac{1}{6c^2+1} \ge \frac{9}{5}$$
$$ \Leftrightarrow \frac{6c(24c^2-25c+4)}{25(6c^2+1)} \ge 0$$
Bất đẳng thức cuối luôn đúng do $0 \le c \le \frac{1}{12}$.
Hoàn tất chứng minh.
Bất đẳng thức xảy ra khi $a=b=c=\frac{1}{3}$ hoặc $a=b=\frac{1}{2}, c=0$ và các hoán vị.

Bài viết đã được chỉnh sửa nội dung bởi tr2512: 20-05-2018 - 10:00


#284
Khoa Linh

Khoa Linh

    Thiếu úy

  • Thành viên
  • 601 Bài viết

Bài 129: $a, b, c \geqslant 0, a+b+c=1$

Chứng minh: $\frac{1}{1+6a^{2}} + \frac{1}{1+6b^{2}}+\frac{1}{1+6c^{2}} \geqslant \frac{9}{5}$

 

Ta viết BĐT lại thành: $\sum \frac{25a^2}{6a^2+1}\leq 5$

Ta có: $6a^2+1=6a^2+(a+b+c)^2=2(2a^2+bc)+3a^2+b^2+c^2+2a(b+c)=2(2a^2+bc)+3a^2+b^2+c^2+2a(1-a)=2(2a^2+bc)+(a^2+b^2+c^2)+2a$

 

Áp dụng Cauchy - Schwarz ta có: 

$\sum \frac{(2a+a+2a)^2}{2(2a^2+bc)+(a^2+b^2+c^2)+2a}\leq 2\sum \frac{a^2}{2a^2+bc}+\sum \frac{a^2}{a^2+b^2+c^2}+2(a+b+c) \leq 5$

Suy ra đpcm. 


Bài viết đã được chỉnh sửa nội dung bởi Khoa Linh: 18-05-2018 - 20:17

$\sqrt[LOVE]{MATH}$

"If I feel unhappy, I do mathematics to become happy. If I am happy, I

 

do mathematics to keep happy" - Alfréd nyi 


#285
quynhanhlh7

quynhanhlh7

    Binh nhất

  • Thành viên mới
  • 21 Bài viết

130. Cho a,b,c là các số dương thỏa mãn ab+bc+ca=3abc

CMR: $\frac{1}{\sqrt{a^{3} +b}}$ + $\frac{1}{\sqrt{b^{3}+c}}$ + $\frac{1}{\sqrt{c^{3}+a}}$ $\leq \frac{3}{^{\sqrt{2}}}$



#286
PhanThai0301

PhanThai0301

    Trung sĩ

  • Thành viên
  • 167 Bài viết

Bài 131: CMR với a, b, c dương thỏa mãn a + b + c = 3 thì

                         $\frac{a^{2}b}{2a+b}+\frac{b^{2}a}{2b+c}+\frac{c^{2}a}{2c+a}\leq 1$.

Bài 132: Cho a, b, c là các số thực tùy ý. Chứng minh rằng:

                        $\frac{ab}{c^{2}}+\frac{bc}{a^{2}}+\frac{ca}{b^{2}}\geq \frac{1}{2}(\frac{a+b}{c}+\frac{b+c}{a}+\frac{c+a}{b})$.


Bài viết đã được chỉnh sửa nội dung bởi PhanThai0301: 24-05-2018 - 23:04

"IF YOU HAVE A DREAM TO CHASE,NOTHING NOTHING CAN STOP YOU"_M10

                                                                                                            


#287
BurakkuYokuro11

BurakkuYokuro11

    Thượng sĩ

  • Thành viên
  • 230 Bài viết

Khuấy đảo topic lại nào . Mình xin gửi tặng topic 1 bài : 
Bài 133 : Cho a,b,c là các số thực dượng thỏa mãn ab+bc+ac 
$\leq$ 3abc .CMR :
     
$\sqrt{\frac{a^{2}+b^{2}}{a+b}} + \sqrt{\frac{c^{2}+b^{2}}{c+b}}+ \sqrt{\frac{a^{2}+c^{2}}{a+c}} +3 \leq \sqrt{2}(\sqrt{a+b}+\sqrt{c+b}+\sqrt{a+c})$


Bài viết đã được chỉnh sửa nội dung bởi BurakkuYokuro11: 22-05-2018 - 00:09

WangtaX

 


#288
Lao Hac

Lao Hac

    Thượng sĩ

  • Thành viên
  • 279 Bài viết

Bài 131: CMR với a, b, c dương thỏa mãn a + b + c = 3 thì

                         $\frac{a^{2}b}{2a+b}+\frac{b^{2}a}{2b+c}+\frac{c^{2}a}{2c+a}\leq 1$.

 

Bài 131:

$\frac{a^2b}{2a+b}+\frac{b^2c}{2b+c}+\frac{c^2a}{2c+a}$

BĐT trên $= \frac{a^2b}{a+a+b}+\frac{b^2c}{b+b+c}+\frac{c^2a}{c+c+a}$

$\leq \frac{a^2b}{3\sqrt[3]{a^2b}}+\frac{b^2c}{3\sqrt[3]{b^2c}}+\frac{c^2a}{3\sqrt[3]{c^2a}}$

$= \frac{\sqrt[3]{a^2b}^2+\sqrt[3]{b^2c}^2+\sqrt[3]{c^2a}^2}{3}$ = $\frac{\sqrt[3]{a^4b^2}+\sqrt[3]{b^4c^2}+\sqrt[3]{c^4a^2}}{3}$

Có: $\sqrt[3]{a^4b^2}+\sqrt[3]{b^4c^2}+\sqrt[3]{c^4a^2}$  

$\leq \frac{a^2+ab+ab}{3}+\frac{b^2+cb+cb}{3}+\frac{c^2+ac+ac}{3}$

$=\frac{(a+b+c)^2}{3}=\frac{3^2}{3}=3$

Vậy $\frac{\sqrt[3]{a^4b^2}+\sqrt[3]{b^4c^2}+\sqrt[3]{c^4a^2}}{3}\leq \frac{3}{3}=1$

Vậy $\frac{a^{2}b}{2a+b}+\frac{b^{2}a}{2b+c}+\frac{c^{2}a}{2c+a}\leq 1$

$Q.E.D$

( Cảm ơn anh MoMo123 và anh Korkot đã sửa giúp em :) )

Bài viết đã được chỉnh sửa nội dung bởi Lao Hac: 01-06-2018 - 08:04

:P


#289
Lao Hac

Lao Hac

    Thượng sĩ

  • Thành viên
  • 279 Bài viết

BÀI 134

Cho $a,b,c$ dương

CMR: 

$\frac{a}{\sqrt{a^2+8bc}}+\frac{b}{\sqrt{b^2+8ac}}+\frac{c}{\sqrt{c^2+8ba}}\geq 1$


Bài viết đã được chỉnh sửa nội dung bởi Lao Hac: 31-05-2018 - 22:33

:P


#290
MoMo123

MoMo123

    Sĩ quan

  • Điều hành viên THCS
  • 334 Bài viết

Khuấy đảo topic lại nào . Mình xin gửi tặng topic 1 bài : 
Bài 133 : Cho a,b,c là các số thực dượng thỏa mãn ab+bc+ac 
$\leq$ 3abc .CMR :
     
$\sqrt{\frac{a^{2}+b^{2}}{a+b}} + \sqrt{\frac{c^{2}+b^{2}}{c+b}}+ \sqrt{\frac{a^{2}+c^{2}}{a+c}} +3 \leq \sqrt{2}(\sqrt{a+b}+\sqrt{c+b}+\sqrt{a+c})$

Đây là lời giải của HelpMelmDying, trình bày ra vậy

Ta có $\sqrt{\frac{a^2+b^2}{a+b}} +\sqrt{\frac{2ab}{a+b}} \leq \sqrt{2(a+b)}$

Tương tự các biến còn lại nên ta được 

$$\sum \sqrt{\frac{a^2+b^2}{a+b}}+\sqrt{\frac{2ab}{a+b}} \leq \sum \sqrt{2(a+b)}$$

Cần chứng minh $\sum \sqrt\frac{2ab}{a+b} \geq 3$

Ta có $\sum\sqrt\frac{2ab}{a+b}=\sqrt{2abc}(\sum \frac{1}{\sqrt{c(a+b)}})\geq\sqrt{2abc}(\frac{9}{\sum\sqrt{a(b+c)}})\geq \sqrt{2abc}{\frac{9}{\sqrt{6(ab+bc+ca)}}}=3$


Bài viết đã được chỉnh sửa nội dung bởi MoMo123: 26-05-2018 - 22:32


#291
anhduc2912

anhduc2912

    Lính mới

  • Thành viên mới
  • 9 Bài viết

bài 130

 

130. Cho a,b,c là các số dương thỏa mãn ab+bc+ca=3abc

CMR: $\frac{1}{\sqrt{a^{3} +b}}$ + $\frac{1}{\sqrt{b^{3}+c}}$ + $\frac{1}{\sqrt{c^{3}+a}}$ $\leq \frac{3}{^{\sqrt{2}}}$

từ giả thiết suy ra$\frac{1}{a}+\frac{1}{b}+\frac{1}{c}=3$

áp dụng bất đẳng thức cauchy cho 2 số dương $a^{3}+b\geq 2a\sqrt{ab} \Rightarrow \frac{1}{\sqrt{a^{3}+b}}\leq \frac{1}{\sqrt{$\frac{1}

lại có {\sqrt{2a\sqrt{ab}}}=\frac{\sqrt{2a\sqrt{ab}}}{{2a\sqrt{ab}}}$$

áp dụng bất đẳng thức cauchy cho a và $\sqrt{ab}$

suy ra  $a+\sqrt{ab}\geqslant 2\sqrt{a\sqrt{ab}} \Rightarrow \frac{2\sqrt{a\sqrt{ab}}}{2\sqrt{2}a\sqrt{ab}}\leq \frac{1}{2\sqrt{2}}\cdot \frac{a+\sqrt{ab}}{a\sqrt{ab}}= \frac{1}{2\sqrt{2}}\cdot \left ( \frac{1}{\sqrt{ab}}+\frac{1}{a} \right )$

tương tự ta được về trái bất đẳng thức sẽ $\leq \frac{1}{2\sqrt{2}}(\frac{1}{a}+\frac{1}{b}+\frac{1}{c}+\frac{1}{\sqrt{ab}}+\frac{1}{\sqrt{bc}}+\frac{1}{\sqrt{ca}})\leq \frac{1}{2\sqrt{2}}(\frac{2}{c}+\frac{2}{a}+\frac{2}{b})=\frac{3}{\sqrt{2}}(dpcm)$

dấu bằng xảy ra khi và chỉ khi a=b=c=1


Bài viết đã được chỉnh sửa nội dung bởi anhduc2912: 26-05-2018 - 23:17


#292
Unrruly Kid

Unrruly Kid

    Trung sĩ

  • Thành viên
  • 113 Bài viết

134) $(\sum a^{\frac{4}{3}})^{2}-a^{\frac{8}{3}}=(b^{\frac{4}{3}}+c^{\frac{4}{3}})(2a^{\frac{4}{3}}+b^{\frac{4}{3}}+c^{\frac{4}{3}})\geq 2(bc)^{\frac{4}{3}}.4a^{\frac{2}{3}}.bc^{\frac{1}{3}}$

$\Rightarrow (\sum a^{\frac{4}{3}})^{2}\geqslant a^{\frac{8}{3}}+8bc.a^{\frac{2}{3}}=a^{\frac{2}{3}}(a^{2}+8bc)$

$\Rightarrow \frac{a}{\sqrt{a^{2}+8bc}}\geqslant \frac{a^{\frac{4}{3}}}{\sum a^{\frac{4}{3}}}$

Thiết lập các bđt tương tự suy ra điều phải chứng minh


Bài viết đã được chỉnh sửa nội dung bởi Unrruly Kid: 27-05-2018 - 16:43

Đôi khi ngươi phải đau đớn để nhận thức, vấp ngã để trưởng thành, mất mát để có được, bởi bài học lớn nhất của cuộc đời được dạy bằng nỗi đau.

#293
PhanThai0301

PhanThai0301

    Trung sĩ

  • Thành viên
  • 167 Bài viết

BÀI 134

CMR: 

A=$\frac{a}{\sqrt{a^2+8bc}}+\frac{b}{\sqrt{b^2+8ac}}+\frac{c}{\sqrt{c^2+8ba}}\geq 1$

  Cách 1: Sử dụng BĐT Holder cách này đã trình bày nhiểu trên diễn dàn.

  Cách 2: Ta chọn số dương r sao cho:

                     $\frac{a}{\sqrt{a^{2}+bc}}\geq \frac{a^{2r}}{a^{2r}+2(bc)^{r}}$

                BĐT tương đương với $a^{2}(a^{2r}+2(bc)^{r})^{2}\geq (a^{2}+bc)a^{4r}$.

                                           <=> $b^{2r}c^{2r}+a^{2r}b^{r}c^{r}\geq 2a^{4r-2}bc$.   

                Áp dụng BĐT AM-GM ta có:

                          $b^{2r}c^{2r}+a^{2r}b^{r}c^{r}\geq 2a^{r}b^{\frac{3r}{2}}c^{\frac{3r}{2}}$.

                Để dấu "=" thì $r=4r-2;3r=2$ <=> $r=\frac{2}{3}$.

                Áp dụng BĐT AM-GM ta có:

                          $A\geq \frac{a^{2r}+b^{2r}+c^{2r}}{a^{2r}+b^{2r}+c^{2r}}=1$ (Q.E.D)

               Dấu bằng xảy ra khi <=> a = b = c.


Bài viết đã được chỉnh sửa nội dung bởi PhanThai0301: 27-05-2018 - 18:12

"IF YOU HAVE A DREAM TO CHASE,NOTHING NOTHING CAN STOP YOU"_M10

                                                                                                            


#294
Lao Hac

Lao Hac

    Thượng sĩ

  • Thành viên
  • 279 Bài viết

  Cách 1: Sử dụng BĐT Holder cách này đã trình bày nhiểu trên diễn dàn.

  Cách 2: Ta chọn số dương r sao cho:

                     $\frac{a}{\sqrt{a^{2}+bc}}\geq \frac{a^{2r}}{a^{2r}+2(bc)^{r}}$

                BĐT tương đương với $a^{2}(a^{2r}+2(bc)^{r})^{2}\geq (a^{2}+bc)a^{4r}$.

                                           <=> $b^{2r}c^{2r}+a^{2r}b^{r}c^{r}\geq 2a^{4r-2}bc$.   

                Áp dụng BĐT AM-GM ta có:

                          $b^{2r}c^{2r}+a^{2r}b^{r}c^{r}\geq 2a^{r}b^{\frac{3r}{2}}c^{\frac{3r}{2}}$.

                Để dấu "=" thì $r=4r-2;3r=2$ <=> $r=\frac{2}{3}$.

                Áp dụng BĐT AM-GM ta có:

                          $A\geq \frac{a^{2r}+b^{2r}+c^{2r}}{a^{2r}+b^{2r}+c^{2r}}=1$ (Q.E.D)

               Dấu bằng xảy ra khi <=> a = b = c.

Mình có cách khác hay hơn cho bài này :icon6:

$<=>\frac{a^2}{a\sqrt{a^2+8bc}}+\frac{b^2}{b\sqrt{b^2+8ac}}+\frac{c^2}{c\sqrt{c^2+8ba}}$

$\geq \frac{(a+b+c)^{2}}{{a\sqrt{a^2+8bc}}+{b\sqrt{b^2+8ac}}+{c\sqrt{c^2+8ba}}}$ ($bunhiacovxki$)

Đặt $M={a\sqrt{a^2+8bc}}+{b\sqrt{b^2+8ac}}+{c\sqrt{c^2+8ba}}$

$={\sqrt{a}.\sqrt{a}\sqrt{a^2+8bc}}+{\sqrt{b}.\sqrt{b}\sqrt{b^2+8ac}}+{\sqrt{c}.\sqrt{c}\sqrt{c^2+8ba}}$

${\sqrt{a}\sqrt{a^3+8bca}}+{\sqrt{b}\sqrt{b^3+8bca}}+{\sqrt{c}\sqrt{c^3+8bca}}$

$M^2\leq (a+b+c)(a^3+b^3+c^3+24abc)=(a+b+c)(a^3+b^3+c^3+3.(2\sqrt{a}\sqrt{b}).(2\sqrt{b}\sqrt{c}).(2\sqrt{c}\sqrt{a}))$ ( $bunhiacovxki$)

$\leq (a+b+c)(a^3+b^3+c^3+3(a+b)(b+c)(c+a))$ ( $cauchy$)

$=(a+b+c)(a+b+c)^{3}=(a+b+c)^{4}$

$=> M \leq (a+b+c)^2$

$A\geq \frac{(a+b+c)^2}{(a+b+c)^2}=1$

$Q.E.D$


Bài viết đã được chỉnh sửa nội dung bởi Lao Hac: 27-05-2018 - 20:02

:P


#295
PhanThai0301

PhanThai0301

    Trung sĩ

  • Thành viên
  • 167 Bài viết

Mình có cách khác hay hơn cho bài này :icon6:

$<=>\frac{a^2}{a\sqrt{a^2+8bc}}+\frac{b^2}{b\sqrt{b^2+8ac}}+\frac{c^2}{c\sqrt{c^2+8ba}}$

$\geq \frac{(a+b+c)^{2}}{{a\sqrt{a^2+8bc}}+{b\sqrt{b^2+8ac}}+{c\sqrt{c^2+8ba}}}$ ($bunhiacovxki$)

Đặt $M={a\sqrt{a^2+8bc}}+{b\sqrt{b^2+8ac}}+{c\sqrt{c^2+8ba}}$

$={\sqrt{a}.\sqrt{a}\sqrt{a^2+8bc}}+{\sqrt{b}.\sqrt{b}\sqrt{b^2+8ac}}+{\sqrt{c}.\sqrt{c}\sqrt{c^2+8ba}}$

${\sqrt{a}\sqrt{a^3+8bca}}+{\sqrt{b}\sqrt{b^3+8bca}}+{\sqrt{c}\sqrt{c^3+8bca}}$

$M^2\leq (a+b+c)(a^3+b^3+c^3+24abc)=(a+b+c)(a^3+b^3+c^3+3.(2\sqrt{a}\sqrt{b}).(2\sqrt{b}\sqrt{c}).(2\sqrt{c}\sqrt{a}))$ ( $bunhiacovxki$)

$\leq (a+b+c)(a^3+b^3+c^3+3(a+b)(b+c)(c+a))$ ( $cauchy$)

$=(a+b+c)(a+b+c)^{3}=(a+b+c)^{4}$

$=> M \leq (a+b+c)^2$

$A\geq \frac{(a+b+c)^2}{(a+b+c)^2}=1$

$Q.E.D$

  Bạn ơi nếu a, b, c =0 thì tất cả cm của bạn đều sụp đổ, cái này đã có nhiều người nhầm lẫn.


"IF YOU HAVE A DREAM TO CHASE,NOTHING NOTHING CAN STOP YOU"_M10

                                                                                                            


#296
Lao Hac

Lao Hac

    Thượng sĩ

  • Thành viên
  • 279 Bài viết

  Bạn ơi nếu a, b, c =0 thì tất cả cm của bạn đều sụp đổ, cái này đã có nhiều người nhầm lẫn.

Không được đâu bạn. Nếu $a=b=c=0$ thì phân số không có nghĩa ( do mẫu số sẽ = 0 ) nên chuyện đó không xảy ra được từ đầu rồi


:P


#297
PhanThai0301

PhanThai0301

    Trung sĩ

  • Thành viên
  • 167 Bài viết

Không được đâu bạn. Nếu $a=b=c=0$ thì phân số không có nghĩa ( do mẫu số sẽ = 0 ) nên chuyện đó không xảy ra được từ đầu rồi

 Ko có gì là ko thể xảy ra cái này là phần lưu ý khi cm BĐT rồi bạn.


"IF YOU HAVE A DREAM TO CHASE,NOTHING NOTHING CAN STOP YOU"_M10

                                                                                                            


#298
MoMo123

MoMo123

    Sĩ quan

  • Điều hành viên THCS
  • 334 Bài viết

Cháy lên nào TOPIC ơi

$\boxed{\text{Bài 135}}$ Cho $a,b,c$ là các số thực dương. Chứng minh rằng

$\frac{8}{a+b)^2+4abc}+\frac{8}{(b+c)^2+4abc}+\frac{8}{(c+a)^2+4abc} +a^2+b^2+c^2 \geq \frac{8}{a+3}+\frac{8}{b+3}+\frac{8}{c+3}$

$\boxed{\text{Bài 136}}$ Cho $x,y,z,t$ là các số thực dương. Chứng minh

$(x^2+1)(y^2+1)(z^2+1)(t^2+1)\geq \frac{16}{27}(x+y+z+t)^2$

$\boxed{\text{Bài 137}}$ Cho $a,b,c>0$ thỏa mãn $a+b+c=1$

Tìm Min $P=2(a^2b+b^2c+c^2a)+a^2+b^2+c^2+4abc$

 



#299
Khoa Linh

Khoa Linh

    Thiếu úy

  • Thành viên
  • 601 Bài viết

Cháy lên nào TOPIC ơi

$\boxed{\text{Bài 135}}$ Cho $a,b,c$ là các số thực dương. Chứng minh rằng

$\frac{8}{a+b)^2+4abc}+\frac{8}{(b+c)^2+4abc}+\frac{8}{(c+a)^2+4abc} +a^2+b^2+c^2 \geq \frac{8}{a+3}+\frac{8}{b+3}+\frac{8}{c+3}$

 

Ta có:

$\frac{8}{(a+b)^2+4abc}+\frac{a^2+b^2}{2}\geq \frac{8}{(a+b)^2(c+1)}+\frac{(a+b)^2}{4}\geq \frac{4}{\sqrt{2(c+1)}}\geq \frac{8}{c+3}$

Thiết lập các BĐT còn lại ta có đpcm


$\sqrt[LOVE]{MATH}$

"If I feel unhappy, I do mathematics to become happy. If I am happy, I

 

do mathematics to keep happy" - Alfréd nyi 


#300
xuanhoan23112002

xuanhoan23112002

    Trung sĩ

  • Thành viên
  • 103 Bài viết

Bài 138: Với a, b, c là các số thực dương. Tìm giá trị nhỏ nhất của biểu thức:

$P=\frac{2}{a+\sqrt{ab}+\sqrt[3]{abc}}-\frac{3}{\sqrt{a+b+c}}$


Bài viết đã được chỉnh sửa nội dung bởi xuanhoan23112002: 29-05-2018 - 13:46






Được gắn nhãn với một hoặc nhiều trong số những từ khóa sau: bất đẳng thức, holder, cosi, bunhiacopxki

5 người đang xem chủ đề

0 thành viên, 5 khách, 0 thành viên ẩn danh